5) A submarine is cruising at -40 meters (40 meters below the surface). It
descends 20 meters, then rises 35 meters. What is the submarine's new depth? *
-25 m
-95 m
25 m
15 m

Answers

Answer 1

Answer:

-25

Step-by-step explanation:

You begin with -40, then decrease by 20 so your new value is -60, then increase by 35 so your final value is -25m... hope this helps (:


Related Questions

i need help its urgent its due today geometry

Answers

Rays, vertex, the vertex of the angle,BXC

Which property is used in the problem below?

2 (x + 4) = 2 x + 8

the associative property
the commutative property
the distributive property
the additive identity property

Answers

Answer:

Distributive property

Step-by-step explanation:

2(x+4)

We can write it as:-

=>2(x)+2(4)

=>2x+8

Answer:

the awnser is c the distributive proporties

Step-by-step explanation:

Which expression has the same value as -18-(-9)?
0 - 18+2
-12-(-3)
- 1945
O-8-(-4)

Answers

Answer:

the answer is -12-(-3)

Step-by-step explanation:

The expression equivalent to -18-(-9) is -12-(-3)

What are expressions?

An expression in maths is a sentence with a minimum of two numbers or variables and at least one maths operation.

Given is an expression, -18-(-9), we are given to find the expression equivalent to it,

-18-(-9) = -18+9 = -9

1) -18+2 = -16 (not equivalent)

2) -12-(-3) = -12+3 = -9

Therefore, we see, the value of expressions -12-(-3) and -18-(-9) are equivalent,

Hence, the expression equivalent to -18-(-9) is -12-(-3)

For more references on expressions, click;

https://brainly.com/question/14083225

#SPJ2

What is the smallest positive number that is prime and 10 less than a perfect square?

Answers

Answer:

The problem states that the answer cannot be a perfect square or have prime factors less than $50$. Therefore, the answer will be the product of at least two different primes greater than $50$. The two smallest primes greater than $50$ are $53$ and $59$. Multiplying these two primes, we obtain the number $3127$, which is also the smallest number on the list of answer choices. So we are done, and the answer is $\boxed{\textbf{(A)}\ 3127}$.

Step-by-step explanation:

Answer:

71

Step-by-step explanation:

I just did the AOPS question, you can see the attachment down below.

Hope this helped! :)

Simple geometry equation

Answers

Answer:

106 or 4x+18  

Step-by-step explanation:

Since Ray QR is a bisector you can determine that PRQ and QRS are equal angles (def. of a bisector). Then, what I chose to do is graph them and find where they intercept which was 22. We now know 22 is x so we can substitute it into the equation and solve to deterine that our answer is 106.

22 = x, explanation:
x+ 31 = 3x-13
-x. -x
31=2x-13
+13.. +13
44-=2x
—. .—
2. 2
22= x

What is the solution to this equation?
X-9 = 17
O A. x = 28
O B. x = 12
O c. x = 8
O D. x = 26

Answers

Answer:

Step-by-step explanation:

x -  9 = 17

x = 26

the answer is D

Find the diameter of a cone that has a volume of 83.74 cubic inches and a height of 5 inches

Answers

Answer:

V = πr²(h/3)

Step-by-step explanation:

83.74 = πr²(5/3)

R = 4.00 in

Diamater = 2r = 2 x 4.00 = 8 inches

Jodi is considering taking online classes at two websites, LearnCenter and EduWorld. Each site requires that students pay a base membership fee, plus they charge a rate per class. The two graphs show the cost for taking online classes through the two websites. A graph titled Cost to Take Classes at LearnCenter has number of classes on the x-axis and cost in dollars on the y-axis. A line goes through points (0, 50) and (1, 70). A graph titled Cost to Take Classes at EduWorld has number of classes on the x-axis and cost in dollars on the y-axis. A line goes through points (0, 20) and (1, 60).

Answers

Answer:

B-LearnCenter charges a higher membership fee but a lower rate per class.

Step-by-step explanation:

Answer:B-learning for ever

Step-by-step explanation:

NEED HELP ASAP
given m||n find the value of x

Answers

The alternate interior angle theorem states that when two parallel lines are cut by a transversal, the resulting alternate interior angles are congruent.

This means that [tex]6x-8=5x+14[/tex]

[tex]x=22[/tex]

Hope this helps. 頑張って!

if a cubic container has a side length of 50.0cm, what is the volume in liter?

Answers

Answer:

The answer is

125 litres

Step-by-step explanation:

To find the volume in liter we must first find the volume in cm³

Since the container is cubic

Volume = l³

where l is the length of one side

From the question

l = 50.0 cm

Volume = 50³

= 125000 cm³

Next we use the conversion

1cm³ = 0.001 L

If 1 cm³ = 0.001 L

Then 125,000 cm³ = 125,000 × 0.001 L

We have the final answer as

125 litres

Hope this helps you

Using the volume of a cube, and conversion of units, it is found that the volume of the cube is of 125 liters.

---------------------

The volume of a cube of side s is given by:

[tex]V = s^3[/tex]

---------------------

Side length of 50cm means that [tex]s = 50[/tex].

Thus, the volume, in cubic centimetres, is given by:

[tex]V = 50^3 = 125000[/tex]

---------------------

To convert the volume from cubic centimetres to liters, it is divided by 1000.

Then:

[tex]\frac{125000}{1000} = 125[/tex]

The volume of the cube is of 125 liters.

A similar problem is given at https://brainly.com/question/23514867

You borrow $350 from your aunt and agree to repay her $400 ($350 principal + $50 interest) in 18 months. What interest rate (using simple interest, and to the nearest tenth) are you paying?

Answers

Answer:

The interest rate is 0.79% per month.

Step-by-step explanation:

Present value of borrowed amount = $350

Future value of borrowed amount = $400

Interest amount = $50

Time duration = 18 months or 1.5 years

Now we have to find the rate of interest by using the above information. Here below is the calculation of interest rate.

Interest rate =  P×R×T

50 = 350 × R×18

R = 0.007936

Or, R = 0.79% per month.

find 3+root 2/3-root 2=a+b root2

Answers

Answer:

a = [tex]\frac{11}{7}[/tex] ; b = [tex]\frac{6}{7}[/tex]

Step-by-step explanation:

[tex]\frac{3 + \sqrt{2}}{3 - \sqrt{2} } = a + b\sqrt{2} \\\\[/tex]

Rationalising [tex]\frac{3 + \sqrt{2}}{3 - \sqrt{2} }[/tex] gives :-

[tex]\frac{3 + \sqrt{2}}{3 - \sqrt{2} } = \frac{(3 + \sqrt{2})(3 + \sqrt{2})}{(3 - \sqrt{2})(3 + \sqrt{2}) } = \frac{(3 + \sqrt{2})^2}{3^2 - (\sqrt{2})^2 } = \frac{11 +6\sqrt{2} }{7}[/tex]

Comparing [tex]\frac{11 + 6\sqrt{2} }{7}[/tex] with [tex]a + b\sqrt{2}[/tex] gives

a = [tex]\frac{11}{7}[/tex] & b = [tex]\frac{6}{7}[/tex]

Answer:

[tex]\frac{11}{7}[/tex] + [tex]\frac{6}{7}[/tex] [tex]\sqrt{2}[/tex]

Step-by-step explanation:

Given

[tex]\frac{3+\sqrt{2} }{3-\sqrt{2} }[/tex]

Multiply the numerator/ denominator by the conjugate of the denominator.

The conjugate of 3 - [tex]\sqrt{2}[/tex] is 3 + [tex]\sqrt{2}[/tex] , thus

= [tex]\frac{(3+\sqrt{2})(3+\sqrt{2}) }{(3-\sqrt{2})(3+\sqrt{2}) }[/tex] ← expand numerator/ denominator using FOIL

= [tex]\frac{9+6\sqrt{2}+2 }{9-2}[/tex]

= [tex]\frac{11+6\sqrt{2} }{7}[/tex]

= [tex]\frac{11}{7}[/tex] + [tex]\frac{6}{7}[/tex] [tex]\sqrt{2}[/tex] ← in the form a + b[tex]\sqrt{2}[/tex]

with a = [tex]\frac{11}{7}[/tex] and b = [tex]\frac{6}{7}[/tex]

Find the factors of the following numbers.
a) 15
b) 22
c) 34
d) 45
e) 50

Answers

Answer:

1,3,5,15

1,2,11,22

1,2,17,34

1,3,5,9,15,45

1,2,5,10,25,50

Step-by-step explanation:

Hope it will help u

Use the table to complete the work to find the missing value. Conversion Chart Pints Ounces 3 48 7 ? 30 480 3 pints 48 ounces 11 pints ? ounces How many ounces are in 11 pints? o 144 o160 o 176 o 192​

Answers

Answer:

176 ounces (C)

Step-by-step explanation:

if you divide 48 by 3 it will give you 16 and there are 11 pints so u wanna multiply 16 and 11 to 176 ounces..

also i got it right on edge :P

Answer: 176

Step-by-step explanation:

27x + 45y and 7(9x + 7y) are these equivalent?

Answers

Answer:

No

Step-by-step explanation:

If you want to know whether they are equivalent or no, we will first have to find the value of both.

[tex]27x+45y[/tex]

Since there aren't any like terms here, your answer would be

[tex]27x+45y[/tex]

Let's solve for next one.

[tex]7(9x+7y)[/tex]

Let's add in parenthesis to 7.

[tex](7)(9x+7y)[/tex]

Now we will separate everything. And also you will have to add.

[tex](7)(9x)+(7)(7y)[/tex]

Now that we added we got our answer.

[tex]63x+49y[/tex]

Both values were not the same, so therefore, your answer is no, they are not equivalent.

Hope this helps!

No

Step-by-step explanation:

If you want to know whether they are equivalent or no, we will first have to find the value of both.

Since there aren't any like terms here, your answer would be

Let's solve for next one.

Let's add in parenthesis to 7.

Now we will separate everything. And also you will have to add.

Now that we added we got our answer.

Both values were not the same, so therefore, your answer is no, they are not equivalent.

Hope this helps!

Read more on Brainly.com - https://brainly.com/question/17379473#readmore

One year, Mr James travels 8 x 10^3 miles for his job. The next year he traveled 1 x 10^4 miles. How many more miles did he travel the second year than he did the first year? Please give me the answer and explain ToT

Answers

Answer:

2 000 miles

Step-by-step explanation:

In the first year, Mr James travels 8 x [tex]10^{3}[/tex] miles = 8 000 miles.

The second year, he traveled 1 x [tex]10^{4}[/tex] miles = 10 000 miles.

Comparing the distance covered in the two years,

10 000 miles - 8 000 miles = 2 000 miles

Therefore, Mr James travels 2 000 more miles in the second year than he did the first year.

Let's assume that Mr James conveys himself to his place of work all through the first year in his private car. But in the second year, there were some days in which the car was not available. So he had to go for his job by public transportation. This itch during those days, could cause an increase in the distance covered from his home to his place of work for the second year.

How much of the circle is shaded? simply your answer

Answers

Answer:

The part that is shaded is 5/14

Step-by-step explanation:

The total circle is 1

1/2 + 1/7 + x = 1  where x is the shaded area

Multiply each side by 14 to get rid of the fractions

14(1/2 + 1/7 + x) = 1*14

7 + 2 + 14x = 14

9+14x = 14

Subtract 9 from each side

14x = 5

Divide by 14

x = 5/14

The part that is shaded is 5/14

Juan pays $15 per hour to rent a paddle boat. He can rent a wetsuit for $5 a day. What amount will he pay if he rents the paddle boat for 6 hours

Answers

Answer:

90

Step-by-step explanation:

15*6=90...

Identify the error in each problem. Explain each error and provide the correction solution. Use at least two complete sentences. PLEASE SOMEONE HELP QUICK ITS DUE TOMORROW AND I WILL MARK YOU BRAINLIEST

Answers

Answer:

the mistake is the addition sign. When a number is next to a parenthesis it means multiplication

Let n be a whole number, and consider the statements below. p: n is a multiple of two. q: n is an even number. Which of the following is equivalent to ~q → ~p? A. ~q → ~p B. q → p C. p → q D. ~p →~

Answers

Answer:

C. p -> q

Step-by-step explanation:

Just did this on Edge2020. Hope this helps :)

Solve the inequality. A. >> O B. x >7 C. x>-5 ) D. x​

Answers

Answer:

A.    [tex] x > \dfrac{7}{3} [/tex]

Step-by-step explanation:

[tex] 8(\dfrac{1}{2} x - \dfrac{1}{4}) > 12 - 2x [/tex]

Distribute the 8 on the left side.

[tex] 4x - 2 > 12 - 2x [/tex]

Add 2x to both sides. Add 2 to both sides.

[tex] 6x > 14 [/tex]

Divide both sides by 6.

[tex] x > \dfrac{14}{6} [/tex]

Reduce 14/6 by dividing the numerator and denominator by 2.

[tex] x > \dfrac{7}{3} [/tex]

3/4 x - 1/2 - 4 = 12

Answers

Answer: x=22

Step-by-step explanation:

Step 1: Simplify both sides of the equation.

Step 2: Add 9/2 to both sides.

Step 3: Multiply both sides by 4/3.

help please will mark brainliest

Answers

Answer:

i can't see that

Step-by-step explanation:

plzz sent me a clear photo

Answer:

1. 1

2. 1  ⋅  10 ^24

3. 1000000000

4. 1

5. 1  ⋅  10 ^20

6. 1  ⋅  10 ^30

7. 1  ⋅  10 ^27

8. 10000

9. 0.01

10. 0.0001

11. 0.1

Hopefully these are right, I simplified all of the problems...sorry if they are wrong

Step-by-step explanation:

I REALLY NEED HELP PLEASE HELP ME

Answers

Answer:

52^8

Step-by-step explanation:

26^8 = 208827064576 + 26

Anyone can help me?​

Answers

Answer:

see the work pictured please

y=-4x - 8
6x + 2y=-10


I need the equation solved by substitution

Please show work :)

Answers

Answer:

x =1.28571428571

Step-by-step explanation:

6x + 2y = -10

y = -4x - 8

6x + 2(4x - 8) = -10

6x + 8x - 8 = -10

14x - 8 = 10

(we put together the like terms)

14x = 10 + 8

14x = 18

x =  [tex]\frac{18}{14}[/tex]

x = 1.28571428571

Jacki evaluated the expression below. 2 cubed (3 minus 1) + 4 (8 minus 12) = 2 cubed (2) + 4 (4) = 8 (2) + 16 = 16 + 16 = 32. What was Jacki’s error? Jacki should have simplified the exponent first. Jacki should have multiplied 4 and 8 first. Jacki did not subtract 12 from 8 correctly. Jacki should not have multiplied

Answers

Answer:

jacki did not subtract 8 minus 12 properly.8 minus 12 equals -4

Answer:

its c

Step-by-step explanation:

on edunuity

The following data points represent the volume of gas in each race car driver's tank (in liters). Sort the data from least to greatest. Find the interquartile range (IQR) of the data set. \text{L}Lstart text, L, end text \text{L}Lstart text, L, end text

Answers

Answer:

its \text{L}Lstart text, L, end text

Step-by-step explanation:

The sorting of the dataset and the information from the five number summary, which includes the three quartiles and the minimum and maximum values indicates that we get;

1. 2.8, 4, 4.3, 6, 7.5, 8.5, 9, 11.6, 12, 12.1

2. The interquartile range is; 7.745

What is the five number summary of interquartile range of a data set?

The five number summary includes the first, second and third quartiles, Q₁, Q₂, and Q₃, and maximum and minimum values of the dataset, which serves to describe the characteristics of the data.

here, we have,

1. The sorted data from least to greatest can be presented as follows;

2.8, 4, 4.3, 6, 7.5, 8.5, 9, 11.6, 12, 12.1

Therefore, the original data in the question is already sorted from least to the greatest

2. The interquartile range, IQR, is the the difference between the third quartile, Q₃, and the first quartiles, Q₁, of the data.

IQR = Q₃ - Q₁

The dataset indicates that we get;

The number of datapoints in the dataset, n = 10

The first quartile = The (N + 1)/4 percentile value

First quartile, Q₁ = (10 + 1)/4 × 10 = The 2.75th value

The 2.75th value is the value 0.75 = 3/4 larger than the 2nd value, which can be found as follows;

Q₁ = The 2.75th value = 4 + (4.3 - 4) × (3/4)  = 4.225

The third quartile, Q₃ = The (N + 1) × (3/40th value, therefore;

Q₃ = The (10 + 1) × (3/4)th value = The 8.25th value (The value 0.25th more than the 8th value )

Q₃ = 11.6 + (12 - 11.6) × 0.25 = 11.7

The interquartile range, IQR = 11.7 - 4.225 = 7.475

The interquartile range = 7.475

Learn more on the interquartile range of a dataset here: brainly.com/question/13119591

#SPJ7

Complete question:

The following data points represent the volume of gas in each race car driver's tank (in liters).

Answer 2 questions about the data set.

1. Sort the data from least to greatest.

2.8

4

4.3

6

7.5

8.5

9

11.6

12

12.1

2. Find the interquartile range (IQR) of the data set.

L

Factorise
a) 2ax + 6ay​

Answers

Answer:

2a(x + 3y)

Step-by-step explanation:

2ax + 6ay

factor out a 2a

2a(x + 3y)

From the expression we can understand that 2a divides into 2ax and 6ay. Therefore, the answer is:
2a(x + 3y)

The area of a sector is 30 m2 in a circle with radius 4 m. What is the arc length of the sector?

Answers

Answer:

15 m

Step-by-step explanation:

The area of the circle is

A =pi r^2

A = pi 4^2 = 16 pi

The area of the sector is 30

The fraction is

30/16 pi  

Take this time 2pi which are the  radians of a circle

30 /16 pi * 2 pi = 15/4

This is the number of radians the angle is

The arc length is s = r * theta where theta is in radians

s = r  theta

  = 4 * 15/4

  = 15

Other Questions
Which is the best description for the graph below? A graph decreases from (0, 3) to (9, 0). The graph increases everywhere. The graph decreases everywhere. The graph remains constant. The graph increases, then decreases. The Johnson family is going to build a new home. They need to have working drawings of their house plans created by a design firm. They will choose between Rock the House Digital Designs and Elegant Home Architects. Rock the House Digital Designs charges $500.00 for the initial consultation with the Johnson family, plus $50.00 for every additional hour of work for the house plans. Elegant Home Architects charge $75.00 per hour for the house plans. Let represent the number of hours for which the Johnson family hires the design firm. Which of the following statements are true? Select all that apply. how can people bring democracy into their lives? 2.7 Code Practice: Question 1Write a program that takes three numbers as input from the user and prints the largestCan someone please help me with this because Ive been struggling all day Minnesota Office Products (MOP) produces three different paper products at its Vaasa lumber plant: Supreme, Deluxe, and Regular. Each product has its own dedicated production line at the plant. It currently uses the following three-part classification for its manufacturing costs: direct materials, direct manufacturing labor, and manufacturing overhead costs. Total manufacturing overhead costs of the plant in July 2014 are $150 million ($15 million of which are fixed). This total amount is allocated to each product line on the basis of the direct manufacturing labor costs of each line. Summary data (in millions) for July 2014 are as follows: Supreme Deluxe Regular Direct material costs $89 $57 $60 Direct manufacturing labor costs $16 $26 $8 Manufacturing overhead costs $48 $78 $24 Units produced 125 150 140 (1) Compute the manufacturing cost per unit for each product produced in July 2014. (2) Suppose that, in August 2014, production was 150 million units of Supreme, 190 million units of Deluxe, and 220 million units of Regular. Why might the July 2014 information on manufacturing costs per unit be misleading when predicting total manufacturing costs in August 2014? If two people answer i will give brainliest to the first one that answers if rightWhat term best classifies cell A? Choose 1 answer: (Choice A) A Plant cell (Choice B) B Fungal cell (Choice C) C Animal cell (Choice D) D Bacterial cell How much energy is passed to the next trophic level in the food chain? Compute the future value of a $130 cash flow for the following combinations of rates and times. (Do not round intermediate calculations. Round your answers to 2 decimal places.)a. r = 7%; t = 10 yearsb. r = 7%; t = 20 yearsc. r = 3%; t = 10 yearsd. r = 3%; t = 20 years I NEED HELP PLEASE !!!! what is 3x-15+6x+2=45 Steve needs to ship a box of dice, each of which is a cube with a volume of 1 cubic centimeter. How many dice can Steve fit in this box? A. 39 B. 270 C. 972 D. 1,944 Describe the novel's style by completing thefollowing statementsThe diction isHis syntax isThe style isThe major was a little man with upturnedmustaches. He had been in the war in Libyaand wore two wound-stripes. He said that ifthe thing went well he would see that I wasdecorated. I said I hoped it would go well butthat he was too kind. I asked him if there wasa big dugout where the drivers could stay andhe sent a soldier to show me. I went with himand found the dugout, which was very good.The drivers were pleased with it and I leftthem there.DONE-A Farewell to Arms, Ernest Hemingway Use the following image to answer the questions below. Murphy had no stock transactions in 2018, so the change in stockholders' equity for 2018 was due to net income and dividends. If dividends were $ 90 comma 000, how much was Murphy's net income for 2018? Use the accounting equation and the statement of retained earnings. A. $ 106 comma 000 B. $ 196 comma 000 C. $ 286 comma 000 D. $ 16 comma 000 Three times the sum of a number and 12. Add. Express your answer as a fraction. Do not include spaces in your answer. 5 1/4 + 15.5 What happened to the U.S. dollar when President Nixon closed the goldwindow?A.it baca me part of a total fiat systemB.it became part of a total commodity systemC.it became part of a partial fiat system D.it became part of a partial commodity system Canada has a population of 1/10 as large as the United States and Canada population is about 32 million about how many people live in United States explain how many zeros in your answer Help with question on Complementary and Supplementary Angles What do you mean by Women's Right?